MQ84 --- Geometry

2013-06-17 4:45 am
MQ84--- GeometryDifficulty:40%Let ABC be a triangle where ∠C=90°. Let M bea point on the segment BC such that AM bisects ∠BAC.Similarly, let N be a point on the segment AC such that BN bisects ∠ABC. Let P and Q be points on AB such that MP and NQ are perpendicular to AB. Find ∠MCN.

回答 (2)

2013-06-17 6:39 am

圖片參考:http://i.imgur.com/HkVk7l9.png

∠MCN
=∠ACB
=∠C
=90° (given)

2013-06-16 23:27:09 補充:
Are you sure that the value of anglePCQ is fixed?
參考: myself :D


收錄日期: 2021-04-13 19:31:27
原文連結 [永久失效]:
https://hk.answers.yahoo.com/question/index?qid=20130616000051KK00284

檢視 Wayback Machine 備份